« first day (33 days earlier)      last day (541 days later) » 
00:00 - 18:0018:00 - 00:00

12:01 AM
0
Q: log(xe^0i)=log(xe^2ipi)?

J.YI am confusing about the differences of the property of natural log in complex analysis and in real region. This question might be a bit stupid, but any answers or explanations of the log properties will be appreciated. The question is: $log(e^{2\pi i})=log|e^{2\pi i}|+iarg(e^{2\pi i})=log|cos(...

0
Q: What is the$Bd(Bd(A))$?

MSEI has a discussion with a friend about the boundary of the boundary, and we do not agree, that's why I ask you the question. For exemple, if $A=[0,1]$, what is $Bd(Bd(A))$ ? To me it's $\{0,1\}$ since $\{0,1\}$ $$Bd(\{0,1\})=Cl(\{0,1\})\backslash Int(\{0,1\})=\{0,1\},$$ but my friend think that i...

0
Q: Terminology conventions

RustyStatisticianI am writing a paper on optimization and was wondering if it is acceptable to say, for example, "There are many local minimums" or should it be phrased to say "There are many local minima"? Are minima and minimums exchangeable vocabulary? Or is there a difference?

 
0
Q: is there a way to bookmark answers?

Steve Yohanani find myself frequenting certain answers repeatedly. is there any means on SO to "bookmark" ones i deem interesting? something akin to "star" on github, perhaps.

 
12:18 AM
0
Q: Young diagram for $S_5$

Omar ShehabI am trying to draw the Young diagram for $S_5$. I know the following pieces of information about $S_5$. The order of the group is $120$. The number of conjugacy classes and so partitions is $7$. Degrees of irreducible representations $1,1,4,4,5,5,6$. The partition is $1 + 10 + 15 + 20 + 20 +...

 
0
Q: Bounty for an early answer after expiration

BerendschotIn a scenario where the question owner aks a 'hard-to-solve' or badly asked question, the amount of views and answers will be most likely very low. In the last case, the chance that one of the answers (or more likely: the only answer) gets more than two upvotes is very low. So the bounty won't be...

 
12:39 AM
0
Q: Find the Values of n and k for which the determinant of the Matrix M(n,k) is Singular

XavierGuéretteI have been stuck on this problem for a couple of days, I don't want the answer, but I would appreciate some help in finding it! Thanks in advance! Consider a Symmetric Square Matrix $M(n,k)$ such that there is $n$ lines and $n$ columns, in which there is $-1$ everywhere expect on the diagonal, ...

 
0
Q: What are some examples of good questions falling under these tags?

Ahmed HusseinWhat are some examples of good questions falling under the following tags? soft-question advice career-development I am asking this question because often, positive reactions from users towards certain off-topic questions allow these questions to get by. Hence, I guess that highly up voted q...

 
12:59 AM
0
Q: Moments and Centers of Mass

mathloverThe moment of inertia is given by equation: $I_L = \lim_{n\to \infty}\sum_{k=1}^n\Delta I_k = lim_{n\to \infty}\sum_{k=1}^nr^2(x_k,y_k,z_k)\delta(x_k,y_k,z_k)\Delta V_k = \iiint_{D}r^2\delta dV.$ What does this equation mean, and what does it signify? Ostensibly, it appears that $\iiint_...

0
Q: laplace of piecewise (possibly dumb question but should have quick answer)

Jacobenter image description here Usually the laplace transforms on piecewise functions are only really defined on one interval or zero on all other intervals, but if it's defined on multiple intervals that means there are two different transforms with two unique answers respective to their intervals...

 
1:12 AM
0
Q: prove $A \times B = B \times A \iff (A=B)\lor(A=\emptyset)\lor(B=\emptyset)$

Amihad to prove the above mentioned iff, but I think I have something missing. would appreciate your advice.

0
Q: Question about the order of the product of two elements in an abelian group

geo17My reference for learning abstract algebra says that in an abelian group $G$ the product of two elements (say $a$ with order $n$ and $b$ with order $m$) has order $mn$ if $gcd(n,m) = 1$. What I don't understand is where the $gcd(m,n)=1$ comes into use in proving this fact? For example, $ a^{mn}...

 
2
Q: Let's boldify some of our code!

SweeperI often run into this problem. When I answer a question, copied some of OP's code and added something, I want to explain why did I add that stuff or what does it do. Often, I need to use words to tell OP where exactly my changes are, but I am extremely bad at expressing this kind of stuff. I thin...

 
1:30 AM
0
Q: Why does $fg = 1/4(f + g) - 1/4(f - g)$ in this problem about integration?

Jerry CrowersCan someone help me understand this? I feel very wrong asking this simple question, but please help me understand this step. Let $f,g$ be real-valued functions on closed interval $I$ on $E^n$. Show that if $f$ and $g$ are integrable on $I$ then so is $fg$. Solution: Notice that $fg = 1/4(f + g)...

0
Q: Richardson's Methods

DoeI need to prove Richardson's Method and the first part of the proof is: Consider the linear system $Ax = b$ where the eigenvalues of $A$ are real and positive. Let $G_{\omega } = I - \omega A$, where $\omega$ is a scalar. Let the iteration be $$x^{n+1} = (I-\omega A)x^{n} +\omega b$$ Show all ...

Short title. Richardson's Methods
-1
Q: Proving that function is a bijection?

Student of MathQuestion on bijection function This is a question on a Final review, and I am having trouble figuring out what to do. Thanks!

Short question. Tagged proof-explanation. Proving that function is a bijection?
0
Q: Constructible decimals?

Sofia JuneSo I have to figure out if $1.23456$ is constructible. I think that it's not constructible since: I know that this is $\frac{123456}{100000}$ so this goes into $\frac{2^6*3*643}{2^55^5}$ and the Fermat Primes are repeated in the denominator so then it's not constructible. Am I overthinking this...

 
2:01 AM
0
Q: When is it ok to post an answer that requires a new R package?

Andrew JacksonI'm new to SO but interested in posting some R solutions where I can. There are potentially lots of solutions to the problems that are posted. When is it appropriate to post an answer that requires a different package than in the original question? How should I provide a code solution showing a n...

 
0
Q: How to detect "Double Peaks" in stock price movements?

slindsey3000Give me an example of how to detect "double peaks" in stocks price movements. Using regular expressions. Is this possible? http://stockcharts.com/school/doku.php?id=chart_school:chart_analysis:chart_patterns:double_top_reversal

0
Q: if f(x)=(x-1)/(x+1) what is the value, in simplest form of f of f(2015)

Universeif f(x)=(x-1)/(x+1) what is the value, in simplest form of f of f(2015); Pretty intresting problem. But attempted a lot but couldnt solve it! Would love your guys help!!

0
Q: What is the real life interpretation of the number of orbits in a bracelet problem?

RobI was asked to find the number of necklaces that can be made using 4 red beads and 6 blue beads. I understand the steps needed to be taken in order to solve this type of problem but I don't quite understand the logic behind it. I know this type of question is essentially asking you for the number...

0
Q: Prove that Lebesgue-Stieltjes outer measure is an outer measure.

Henry WFor any non-decreasing $f: \mathbb{R} \to \mathbb{R}$ and $S \subseteq \mathbb{R}$, The Lebesgue-Stieltjes outer measure associated with $f$ is $$\lambda^*_f(S) = \inf\left\{\sum_{j=0}^\infty (\lim_{x \uparrow b_j}f(x) - \lim_{x \uparrow a_j}f(x)) \mid a_j, b_j \in \mathbb{R} \land ...

0
Q: stability conditions for ODE problem

SmallElephantConsider a scalar problem of the form $$y(t)' = \mu^2 y(t).$$ Derive stability conditions for given problem.

Title contains problem. Short question. Tagged pde, differential-equations. stability conditions for ODE problem
0
Q: Confusing partitions of $S_5$ in two different sources

Omar ShehabI am trying to understand the partitions of $S_5$ created by it's conjugacy classes but two sources have two different partitions. Source 1: Source 2: So, for example, in the first table, the partition for cycle structure ()()()()() i.e. $5$ $1$-cycle is $5+0+0+0+0$ but in the second table i...

Title contains confus. Question contains please. Confusing partitions of $S_5$ in two different sources
0
Q: Realizing a quotient ring as an abelian group

Daniel AkechShow that $\mathcal{M} = \mathbb{Z}[X] \backslash(X^{2} - X, 4X +2)$ is finitely generated abelian group. What is the general procedure for handling these types?

0
Q: Problem on composition two linear maps on a vector space

user296006Let $ P$ be the vector space of all polynomials over $\mathbb R$.Let T and S be two linear maps from $ P$ to itself such that TOS is the identity map.Then which are correct? SOT may not be the identity map. SOT must be identity map, but S and T need not be the identity map. S and T must be the...

0
Q: How do I estimate coefficients of y=αx+βy+γz+ϵ?

ZombI'm using R, and I just wanted to know that how do I calculate residuals for this model? And when I use lsfit for this, why don't I get a value for gamma?

 
2:50 AM
0
Q: Ask for a solution to avoid SE being blocked in China

DanielSorry to bringup this topic again. I read these links but they did not answer my question. Is Stack Overflow accessible in China? What's up with China? It seems to me certain JavaScripts used by Stack Exchange are somehow on the blacklist of our government media control. (I say this because I sa...

 
0
Q: How to determine definite integrals for polar coordinates

igknightonInside the sphere x^2 + y^2 + z^2 = 25 and outside the cylinder x^2 + y^2 = 4 I've so far converted the formula to z = r^2-5r outside the cylinder r=4 For the internal integral, I have the definite integral going from 0 to 4, but the external integral is from 0 to 2pi. I'm confused as to where ...

This site uses MathJax formatting of formulas. More tips here. (autocomment)Normal Human 21 secs ago
0
Q: In need of book recommendations (Soft Question)

VelvetUndergradI tried to label this with as many soft question labels as possible. To give you a gauge of where I am math-wise, I'm a senior in high school who has gone through all the basic subjects (geometry, algebra, trigonometry, basic statistics/probability, calculus I). In addition, I've explored a few ...

Words such as question are uninformative in titles. Please edit the title so that it better describes the specifics of your question. Do not hesitate to make it longer or include a formula if needed. More tips here. (autocomment)Normal Human 21 secs ago
0
Q: How to determine the locus of a complex number

Aditya Bidwaiwe are given that Re[(z+5)/(z-5)]=0. What is the locus of these points? is it a circle or y axis? my professor told me that it is a circle.

0
Q: Is this a valid statement?

socratesLet f:[a,b] -> ℝ be a continuous function and let a<x<y<b. Then f is continuous in [x,y] and differentiable on (x,y). Can anyone tell me which theorem states this?

Short title. Short question. Is this a valid statement?
 
3:16 AM
-2
Q: Reopen votes should be allowed to add comments

Harikrishnan TThis is the question in the context. Regarding the above question, it was marked as duplicate and was closed. But unfortunately the question marked as its duplicate was an entirely different one, which has absolutely no relation with the question in context. The question is about disabling spell...

 
0
Q: How much life does it take to stack your deck? (Sorting problem)

MarsOneRoverThere is a card in Magic the Gathering called Lim-Dul's Vault. While it is slightly more complicated than presented, the question I would like to consider is this: Pay 1 life. Look at the top 5 cards of your deck. You may rearrange these cards in any order. You may repeat step 1. If you do...

 
3:46 AM
0
Q: Approximate a summation

user19405892Approximate $3+ \displaystyle \sum_{i = 2}^{999}\dfrac{3(1000-x)}{1000+x}$. It may help to know that $\ln 2 = 0.69$. I was thinking of doing the integral test to approximate this but I am unsure if this would work.

0
Q: Parameterization of Ellipsoid

portin.danielI have a question asking me to evaluate $\iint_\Sigma \mathbf{F} \cdot \mathbf{n}~dS$, where $\Sigma$ is the lower half of the ellipsoid $z = -2 \sqrt{1 - x^2 - y^2}$ with $\mathbf{n}$ directed upwards. However, I'm having trouble deriving the parametric equations for $\Sigma$. I know the general...

0
Q: Evaluate an integral by changing to polar coordinates.

user2411824I'm having issues with this problem: Evaluate by changing to polar coordinates ∫∫15xdydx The regions of integration are outer: 1/2 to 0 Inner: sqrt(1-x^2) to sqrt(3x) I know that r=1 because solving y=sqrt(1-x^2) gives a circle with radius 1 as the top limit of integration and the lower being ...

This site uses MathJax formatting of formulas. More tips here. (autocomment)Normal Human 21 secs ago
 
0
Q: Is this too vague? : What are the physical limitations that I have to keep in mind, when writing about nanobots?

MalandyIs this too vague? What are the physical limitations that I have to keep in mind, when writing about nanobots? Like maximum effective speed, waste heat minimums, computation ability minimums, etc. Should I move to Worldbuilding, or does this have enough physics content to count as a Ph...

 
4:03 AM
0
Q: How to show that the following is true?

Matt B. How do I go about showing that this following statement is true?

 
4:19 AM
0
Q: A basic question about Series

user270281I was reading the famous "Calculus" by Spivak and at the beginning of the chapter on infinite series, he states: "It's an easy exercise to prove that if both $\displaystyle \sum_{k=1}^{\infty}a_n$ and $\displaystyle \sum_{k=1}^{\infty}b_n$ exist, then $\displaystyle \sum_{k=1}^{\infty}a_n+b_n$=$\...

Words such as question are uninformative in titles. Please edit the title so that it better describes the specifics of your question. Do not hesitate to make it longer or include a formula if needed. More tips here. (from a bot)Normal Human 29 secs ago
 
4:34 AM
0
Q: Truth values Based on some derivatives

PrasanthiFollowing 4 logical propositions are all true : 1. (A → B) ∧ (A → ¬B) 2. ¬A → B 3. ¬(B ∧ D) 4. C ∨ ¬A Truth value of proposition A,B,C,D:

0
Q: Designing a DFA to accept a string

techno I have created the following FA Im i correct?

0
Q: If A norm-2 is less than 1, prove A - I is nonsingular.

John VoHow would I come about this? This is very hard. This question is on my practice final. Any help would be helpful. Also, any tips on proving things?

0
Q: Division of complex numbers :$ \dfrac{(-1+i)^{10}}{(-\sqrt{2}-i\sqrt{2})^{15}} $

sajjadI tried to solve this division of complex numbers and reached to below answer is it true?if not please leave the true answer. $ \dfrac{(-1+i)^{10}}{(-\sqrt{2}-i\sqrt{2})^{15}} $ my solution (first solve the denominator) : $(-\sqrt{2}-i\sqrt{2}):$ $r=\sqrt{2+2}=2$ $\tan(\theta)=\dfrac{-\sqrt{2...

Tall formulas in titles break the layout of question lists. Please replace \dfrac with \frac in the title. (autocomment)Normal Human 24 secs ago
 
5:07 AM
0
Q: Is there any way to check how many reputations I had on a specific date?

StarkeenIs there any way to check how many reputations I had on a specific date in past. Any Stack data sql queries?

 
Short title. self learning maths
0
Q: Linearity of inner product

turtleIs there a way to derive the fact: $\langle cA, C\rangle = c\langle A, C \rangle$ from $\langle cA + B, C \rangle = c \langle A, C \rangle + \langle B, C \rangle$? $\langle \cdot, \cdot \rangle$ is an inner product and $A, B, C$ are vectors and $c$ is a scalar. I tried this: plugging $B = 0$, ...

0
Q: Use the renewal equation to show that the renewal function of a Poisson process

zxc123Use the renewal equation to show that the renewal function of a Poisson process with rate λ > 0 is m(t) = λt.

0
Q: $L^p$ convergence of $f_n$

hl0202Suppose that $f_n\to f$ in $L^p$. Then, is it true that $\int |f_n|^p \to \int |f|^p$? When $p=1$, the result is clearly true since $\int ||f_n|-|f|| \leq \int |f_n - f|$, but how do you show/disprove the result when $p > 1$?

0
Q: Name for a sinc-like function

norioAs far as I understand, with some real number $a$, the function $$f(x) = \frac{\sin(ax)}{ax}$$ is called 'sinc' function. Is there a name for a function like the following one? $$g(x) = \frac{\cos(ax)-1}{ax}$$

0
Q: Suppose $C$ is the unit circle in the plane and $f:C\to C$ is a map not homotopic to the identity, then $f(x)=-x$ for some $x\in C$.

grayQuantThis from "Basic Topology" by Armstrong. I can't figure out what $f(x)=-x$ is doing when mapping from unit circles. Is this the antipodal map?

0
Q: How to expand $|(1-H(f))|^2$

sleeve chenThis problem might be simple. How to expand $|(1-H(f))|^2$? I just see one answer in the following: (A typo: $S_X(f)$ should be outside the $\{\}$). My answer is $1+|H(f)|^2 + 2|H(f)|$. However, "$2\text{Re}[H(f)]\neq2|H(f)|$". (One is the lenght of the vector, the other the length o...

 
0
Q: What (if anything) is wrong with my swapfile question regarding being off-topic?

JDługoszDedicated swap drive without using a drive letter I see this has been downvoted-without-comment, and also has two close votes for being off-topic. How is it any more off-topic than many of the other questions I see tagged with swapfile? Is there some subtlety I'm missing? Just looking at the t...

 
5:35 AM
0
Q: Integrate ( (sin 2x)^3-x(cos x)^3 )/(cos x)^2 dx

AmeetIntegrate ( (sin 2x)^3-x(cos x)^3 )/(cos x)^2 dx. Help me im quite blur and how should i start.a little show on working are gladly appreciate

This site uses MathJax formatting of formulas. More tips here. (from a bot)Normal Human 25 secs ago
0
Q: Question about symmetric group action

QualityI am trying to analyse the following. Assume $S_{3}$ acts on a non empty set $T$, and that is has $3$ orbits. What can we say about the possible cardinalities for the set T? My thoughts: If $G=S_{3}$ then $|G|=3!=6$ and we are consider $G \times T \to T$ , $(g,t) \to g \star t$ I also know t...

Words such as question are uninformative in titles. Please edit the title so that it better describes the specifics of your question. Do not hesitate to make it longer or include a formula if needed. More tips here. (from a bot)Normal Human 24 secs ago
 
0
Q: Same question asked by two user given

ketanToday i show two question Question1 and Question2 which have same content without change of any single word. The Question has answer given by same person and same answer. There is something wrong? What action should i take for this?

 
5:55 AM
0
Q: Solving inequality

bulbasaur Okay I found this one on a test and I am still struggling to solve this one don't know where to start..here it goes $ 1 \le \frac{(3x^2 -7x+8)}{x^2 +1} \le2 $. How do I find the range of x?

Short title. Short question. Solving inequality
 
6:12 AM
0
Q: One sample z-test hypothesis testing with 5% significance level

John SmithA certain cellular device is advertised as being able to last for 7.0 to 9.0 hours of screen-on time. In experiments carried out to test this claim, the time in hours, X, was measured on a random sample of 250 occasions, and the data obtained is summarized by \sum \left ( x-7.6 \right ) = 68.3 an...

Questions tend to get more attention when they have a tag for a broad area of mathematics relevant to the question. Some of these tags might fit. (autocomment)Normal Human 21 secs ago
0
Q: Continuous map between circles

user296029Let $C, C_1, C_2 \subset \mathbb{R}^2$ be circles. Let $f: C \rightarrow C_1 \cup C_2$ be a map such that $f(x_1) \in C_1$ and $f(x_2) \in C_2$. Does it follow that $f$ is not continuous?

 
0
Q: Do not make the review control panel sticky

JazzepiThe control panel that's present when resolving a review should not be sticky. It should just be a normal part of the webpage that you scroll past. Currently it takes up half the space in my page. When trying to edit someone's post the site has given away half of my editor space to a set of cont...

0
Q: I do not need help, I need an answer. Dealing with workarounds

Bruno GriederOn a few occasions, I ask a question and people do not reply to my question, they give me a workaround. The typical flow goes like this Q: Why can I not do this: details ? A: You can do this instead Q: OK, thanks, but why... etc Some of these workarounds are really clever and useful, and sh...

 
6:33 AM
0
Q: What is the derivative of tan(ax+b) with respect to tan(ax+b)?

Sohaib FarooqI tried to solve this question. It is a multiple choice question. First I took the derivative of tan(ax+b) which is sec^2(ax+b) but If I take the derivative with respect to x it will be a sec^2(ax+b). what is find confusing is d/d tan(ax+b) = sec^2 (ax+b) d/d tan(ax+b) *(ax+b) I can't really u...

Welcome to Math.SE, Sohaib Farooq. This site uses MathJax formatting of formulas. More tips here. (autocomment)Normal Human 22 secs ago
0
Q: Map between circles

user296029Let $C_1, C_2 \subset \mathbb{R}^2$ be concentric circles in the plane. Suppose that $C_1$ bounds $C_2$. Let $f: C_1 \rightarrow C_2$ be a map such that for some $y \in C_2$, $f(x) = y$ for all $x \in C_1$ and that $x$ and $y$ must be connected by a path that is entirely contained in the interior...

Short title. Map between circles
0
Q: extending models of zfc

davidpI'm not sure if this question is coherent. But here goes. (1) Let M is a model of ZFC. For which axioms, A, of ZFC is there a model, M', extending M in which A no longer holds? (For instance, I take it that the empty set axiom fails to meet this condition.) (2) For the conjunction of axiom...

0
Q: If $f$ takes Cauchy sequence to Cauchy sequence then $f$ is continuous

AmartyaIf $f:X\to Y$ takes Cauchy sequence to Cauchy sequence then prove that $f$ is a continuous function. Let $x_n$ be a sequence in $X$ such that $x_n\to x\implies x_n$ is Cauchy $\implies f(x_n)$ is Cauchy but that does not guarantee that $f(x_n) \to f(x)$ . So how is the above result true.Pleas...

0
Q: Generalized Woodbury identity

shuohgAs we know, if $A$ and $C$ are not singular, the Woodbury identity is : $(A+UCV)^{−1}=A^{−1}−A^{−1}U(C^{−1}+VA^{−1}U)^{−1}VA^{−1}$. What if $A$ is singular and $C$ is not singular, is there identity (like the Woodbury identity) to calculate the inverse of $A+UCV$ ? Thanks.

 
7:10 AM
0
Q: Prove that the upper and lower integrals, noted below in R^d...

user217189Prove that the upper and lower integrals, noted below in R^d, do not depend on the choice of rectangle R as long as it contains E

0
Q: Generalization on older SE question: not just covering $1$, but all rational numbers.

0.5772156649...Older SE question lies here. So I will change the question such that you can understand the question better: $$\sum_{c\space\subset \Bbb{Co}}\frac{1}{c}\le k$$ -where your goal is to get to $k$ with the desired specific composites -You must add up the specific composites (group $\Bbb{Co}$) su...

 
7:21 AM
0
Q: x^y = exp( ln(x) * y ), not a real solution for decimal numbers?

Simon GoodmanI am trying to understand how to calculate x^y where y is a decimal number, (2^2.3) According to wikipedia, the 'solution' would be x^y = exp( ln(x) * y ) But if we break it down further, z = ln(x) * y x^y = exp( z ) But in that case, z is almost certainly not going to be an round number ...

Welcome to Math.SE, Simon Goodman. This site uses MathJax formatting of formulas. More tips here. (from a bot)Normal Human 21 secs ago
 
7:33 AM
0
Q: Find the eigenvalues?

Chris$\begin{bmatrix}4 & -5 & 1 \\ 1 & 0 & -1\ \\ 0 & 1 & -1\end{bmatrix}$ I usually set 0 equal to $det(A- (lambda * I))$ to find the eigenvalues, but the book says they are 2, 1, and 0, which isn't what I got. Am I missing a step?

0
Q: If I know the sides of right triangle, how can I know its angle?

user283144If I know the sides of right triangle, how can I know its angle? without using calculator

0
Q: How to find the perimeter

Rayan AhmedImage link : http://tinypic.com/r/30wph1j/9 For the above 8 faced geometrical shape where each edge is perpendicular to the adjacent one. How to get the perimeter of the whole face? I tried to draw different line to make triangle and rectangle to get the unknown edge's length. Is it possible...

0
Q: Normed Vector Space

MelodyLet $V$ be a real normed vector space. Suppose that A is an open set from $V$ Show that the set $\frac12A$ = {$\frac12x: x \in A$} is also open. Let V be a complex vector space. A norm on V is a function || · || : V → R that satisfies the following three conditions: (i) ||v|| ≥ 0, ∀ v ∈ V, and ...

Short title. Normed Vector Space
0
Q: Is this number in O(log(n))?

user296012Is this number $\big[log(n) + \sum_{j=1}^n (-log(n+1) - j(log(n)) + 1)\big] \in O(log(n))$?

Short title. Short question. Is this number in O(log(n))?
0
Q: Find all bifurcation values of a function

user3773246Find all bifurcation values of the function : x' = u + cos(x) + cos(2x) How do I find all bifurcation values of u? The solution: u < -2, u = -2, -2 < u < 0, and u = 0. How basically, I need to find the values for which u does not exist. Then I use linearization find the stability. But how?? B...

Welcome to Math.SE, user3773246. This site uses MathJax formatting of formulas. More tips here. (autocomment)Normal Human 24 secs ago
0
Q: Proving Z × N is countable.

AlbertHow would I prove that Z × N is countable? The hint given was to follow to indicated order. Thanks!

Short question. Tagged proof-explanation. Proving Z × N is countable.
0
Q: Domain and range of locus formed from parametric equations

Lee Warrenx = t^2 + 2t y = 4(t+1)^2 Determine the cartesian equation of the locus? What is the domain of the locus?

 
8:13 AM
0
Q: Laurent Serie for csc(z)

Renato Collado TelloI have to find the Laurent series for $$ csc(z) \qquad |z|>0 $$ but I really don't know how to start. Please, guys.

Consider adding a tag for a broader subject area to which the question belongs. Some of these tags might fit. (from a bot)Normal Human 40 secs ago
0
Q: $X=\mathbb{R}^n$ is reflexive. Need help understanding the proof.

user264885I know that every finite dimensional normed space is reflexive, so this would imply X is reflexive. However, the author writes that this also follows directly from the fact that the second dual of $X$ is the space itself. I'm failing to relate how $X = X''$ and reflexivity of are one and the s...

0
Q: Need help with writing proof in elementary Set Theory

DannzI think the best way to prove this is by contradiction, but I'm struggling with the concept of how t write it properly. $$ A\setminus B = \emptyset \leftrightarrow A \subseteq B$$ Thanks in advance.

Welcome to Math.SE, Dannz. Words such as help are uninformative in titles. Please edit the title so that it better describes the specifics of your question. Do not hesitate to make it longer or include a formula if needed. More tips here. (autocomment)Normal Human 26 secs ago
0
Q: Proof Involving Hilbert Space

MelodyLet $0$ be the sequence of real numbers with all the components equal to $0$ and, for each $n \in \Bbb N$, let $\delta_n$ be the sequence of real numbers whose n-th component equals $1$ and all other components are $0$. Show that $0 \in l^2$ and that for any $a \in \Bbb R$, we have that $a\delta...

 
8:35 AM
1
Q: What to do with questions that are likely to be made-up click bait?

WrzlprmftFor example: 17 year old PhD student doesn't want to take credit for his own extraordinary paper – 269 views and +9/−0 votes in 11 hours. My paper was too revolutionary - reviewers at a top journal rejected it "by simply reading the title". What now? – 3141 views and +15/−8 votes in 8 days; clo...

 
0
Q: Problem about $Y(t)=X(t)\text{cos}(2\pi f_0t+\theta)$ with $\theta\sim \text{unif}[-\pi,\pi]$

sleeve chenSuppose given the following: 1. $Y(t)=X(t)\text{cos}(2\pi f_0t+\theta)$ with $\theta \sim\text{unif}[-\pi,\pi]$, and $X(t)$ is a random process. My problem is: If not specified, can we say $X(t)$ and $\text{cos}(2\pi f_0t+\theta)$ uncorrelated? If I want to prove "yes", could I do the fol...

Title contains problem. [Problem about $Y(t)=X(t)\text{cos}(2\pi f_0t+\theta)$ with $\theta\sim \text{unif}[-\pi,\pi]$](math.stackexchange.com/q/1560768)
0
Q: Simple $M/M/1$ service time

OscarI'm trying to understand queueing systems and I found some notes online. They define $\lambda$ as the mean arrival rate, and $\mu$ as the mean service rate (the average number of customers who can be served by a single service station per unit time). The example they give is Customers arrive...

0
Q: characteristic function using conjugate property

Salim Shaikhto prove that e−i|x|e−i|x| is not a characteristic function e−i|x|e−i|x| =cos|x|-i sin|x| Its conjugate will be cos|x|+i sin|x| which is not equal to fi(-x) Is my solution correct ??

0
Q: Constant Gauss curvature from bipolar projections.

NarasimhamPlease help finding z-coordinate for constant positive and negative Gauss curvature in Mongé form : $$ x= \sqrt{R^2 + T^2} + R \cos u ,\, y= R \sin u ,\, z= .. $$ (Earlier seen Cassinian Ovals projection used to build up such forms, this may have been done before.)

This site uses MathJax formatting of formulas. More tips here. (from a bot)Normal Human 42 secs ago
0
Q: Obtaining recursion formula in Maple

JamgreenWhy can't Maple simplify this differential equation further? I want to obtain a recursion formula for $c_{n+1}$. I have tried using collect( ..., c[n+1] ), isolate( ..., c[n+1] ) and solve( ..., c[n+1] ) but it doesn't work. First term on left hand side can be written as $$ \begin{split} 3 \...

0
Q: Eigenvalues of graphs

user296055Given an undirected, unweighted graph $G$, let $A_G$ denote the graph's adjacency matrix. I want to understand in what cases there exists negative eigenvalue for $A_G$, for example, any claim of the form "$G(n,p)$ has at most $x$ negative eigenvalues" would be great for me. If there is any spec...

 
9:18 AM
2
Q: Is it better to edit or to flag, when a simple rephrase of a question would make it on topic?

Michele Giuseppe FaddaEg.: a user asks for "the best solution" to a problem, but from the context you can infer that he/she just needs a solution, a working example. Similarly, the user asks for a "tutorial" (external example) when an example could be given on SO, is quite probably everything the OP needs, and would ...

 
0
Q: Solve ((dy/dx)²+1)(x-y)²=(x+ydy/dx)² .

UtkarshThe question is as follows: Solve ((dy/dx)²+1)(x-y)²=(x+ydy/dx)² . Can we reduce it to Clairaut's form by taking appropriate substitution? Please help.

Short title. Short question. Question contains please. Solve ((dy/dx)²+1)(x-y)²=(x+ydy/dx)² .
 
9:37 AM
0
Q: Can every monad give rise to a monad transformer?

awllower Can every monad give rise to a monad transformer? In the paper Calculating monad transformers with category theory by Oleksandr Manzyuk, one finds a construction of monad transformers as translating monads along adjunctions. In particular, considering the Eilenberg-Moore construction, we c...

Consider adding a tag for a broader subject area to which the question belongs. Some of these tags might fit. (autocomment)Normal Human 28 secs ago
0
Q: properties of limit inferior

Adrian BerteanuProve the following property: "If $x_n<=$y_n for every n>=n_0,then: the limit inferior of $x_n is <= than the limit inferior of $y_n." I'm lost on that.Please,could you give me any help?

Short title. Question contains please. properties of limit inferior
0
Q: Present Value at time 0

Fatima AtavlievaLand is an indestructible capital asset that offers a perpetual revenue flow. You own a piece of land that returns $20,000 per year forever. The relevant real interest rate to consider is 4%. What is the present value of this land at time 0?

Short title. Title ends with a digit. Present Value at time 0
 
9:53 AM
0
Q: Can not reply to any post on Stack Overflow

user2514908I am trying to post response to some of the questions in the post but I am receiving the following error: We are no longer accepting answers from this account. See the Help Center to learn more. I helped a user previously by looking the answer from Google: Windows script to compare folde...

 
10:03 AM
0
Q: How can I see the order of accuracy of an ODE method?

user296066Let's say I have solved an ODE with Euler's forward method, and also solved it using RK4. Is there any way to look at the graph and "see" the order of accuracy of the methods?

0
Q: Use integration by parts to evaluate each function

Fatima AtavlievaI tried to solve this task,but I couldn't do anyway...Please,If you know help me!Thank you! Use integration by parts to evaluate each function ∫𝑥3√(1+𝑥2)𝑑𝑥

Short question. Question contains please. Use integration by parts to evaluate each function
0
Q: Dirac Delta property

Rafał ApriaszI was calculating one physical problem and I stopped at one thing. By definition Dirac delta is given by that expression: $$ \int_{-\infty}^{\infty}\delta(x)dx = 1 $$ and additionaly there is one property which is correct (I sure that, because I've proven it 2 years ago but I can't find a book)....

Short title. Dirac Delta property
0
Q: Deciding when to use $\frac{\pi}{2}-x$ or $\frac{\pi}{2}+x$ and ${\pi}+x$ or $\pi-x$ during integration

pcforgeekHow can I decide when to use $\frac{\pi}{2}-x$ or $\frac{\pi}{2}+x$ and ${\pi}+x$ or $\pi-x$ while solving integration involving trigonometric function?

0
Q: Help proof $ \underset{x\rightarrow0}{lim}f(ax+b)=\underset{x\rightarrow b}{lim}f(x)$

GuestAssume that $\underset{x\rightarrow b}{lim}f(x)$ exists $\forall a,b\in\mathbb{R}$ and $a\neq0$ $ \Rightarrow$ $ \underset{x\rightarrow0}{lim}f(ax+b)=\underset{x\rightarrow b}{lim}f(x)$

0
Q: Prove that $f$ is differentiable at $(0,0)$.

AmartyaDefine $f:\Bbb R^2\to \Bbb R^2$ by $f(x,y)=(x+2y+y^2+|xy|,2x+y+x^2+|xy|)$ Prove that $f$ is differentiable at $(0,0)$. In order to prove that $f$ is differentiable at $(0,0)$ we must have $\lim _{|h|\to 0}\dfrac{|f(h)-f(0)|}{|h|}= 0$ But here $\lim_{h^2+k^2\to 0}\dfrac{h+2k+k^2+|hk|}{h^2+k^2}$...

 
10:31 AM
0
Q: Sketching $y=a^x-a^{2x}, a>0$

DonWhen I need to sketch: $y=a^x-a^{2x}, a>0$, do I need to graph two functions? One when $0<a<1$ and the other when $a>1$? I didn't get any difference in the ascending descending intervals. Because $y'=lna(a^x-2a^{2x})$ so y'=0 when $x=log_a0.5$ and $y''(log_a0.5)<0$ So it has a max point. Than...

0
Q: invariant subspaces endomorphism

tom785I'm wondering about invariant subspaces. If we have an endomorphism $f$ in $\mathbb{R^3}$ such as its matrix in canonical basis is \begin{pmatrix} 1 & 1 & 0 \\ -1 & 2 & 1 \\ 1 & 0 & 1 \\ \end{pmatrix} and $V=Vect{(1,1,1)}$, $Z=Vect((1,0,0),(0,1,-1))$ Is $V$ invariant by $f$ ? And how...

0
Q: $\forall x (x = f(y)) \implies \forall y (y = f(x))$

今天春天I was wondering about the following fact: $\forall x (x = f(y)) \implies \forall y (y = f(x))$ I think this is not true, but I can't find a counterexample but not I can prove it is wrong. Any ideas?

A title should not be all-MathJax; having some plain text helps with search and navigation. (autocomment)Normal Human 21 secs ago
0
Q: How to see this inequality?

SalamoSuppose we have an $(n\times n)$-matrix $A(t)$ and an $n$-vector $b(t)$ that depend continously on $t$. Let $u\colon\mathbb{R}\to\mathbb{R}_{\geq 0}$ some function. In a proof, I found the following inequality: $$ 2\lVert A(t)\rVert u + 2\lVert b(t)\rVert\sqrt{u}\leq(2\lVert A(t)\rVert+\lVert b(...

 
10:49 AM
0
Q: Help finding SO podcast on human or bot

Devraj GadhaviCan someone please help me find SO podcast, where Jeff Attwood was annoyed with a person posting spam questions, by entering captcha values. It was an incident where questions with same title and text were posted numerous times within minutes. And it continued for a while until Jeff and SO team ...

 
0
Q: Calculating Poisson process problem

whoisitA shop sells both hot and cold drinks. Hot drink sales occur at the instants of a Poisson process with expectation 30 drinks per hour.Cold drink sales occur at the instants of a Poisson process with expectation 20 drinks per hour. 60% of customers purchasing a drink are female, 40% of customers p...

Consider adding a tag for a broader subject area to which the question belongs. Some of these tags might fit. (autocomment)Normal Human 21 secs ago
0
Q: Is every polynomial with complex coefficients solvable?

continentalDoes every polynomial $a_0+ a_1x+ ... + x^n = 0$ where $a_i \in \mathbb{C}$ have a solution?

0
Q: limit of a sequence defined recursively, is my proof correct?

RooneySo i have a sequence defined by $a_1 =1$ and $a_{n+1} = a_n + \frac{1}{a_n}$ and i would like to know $\lim_{n\rightarrow \infty} a_n$. I have said that the sequence $a_n$ is unbounded and thus the limit does not exist. But I don't know how to rigorously prove that the sequence is unbounded and i...

Consider replacing (analysis) with a more specific tag for the relevant branch of analysis. (autocomment)Normal Human 29 secs ago
 
11:13 AM
0
Q: What is P(x̄ < 120)

Joakim IsoahoI just want check if I got this right X~N(150, 60^2) N = 500 We take a sample n = 100 from the sample we take the sample mean x̄ What is P(x̄ < 120) ? When I tried doing this I got insanely large Z scores, did I do something wrong?

Short title. What is P(x̄ < 120)
0
Q: Convergence of $\int_0^{1/2}\frac{1}{x^\alpha \log x}dx$

MarkTo establish, for which values of real parameter $\alpha$, the integral $$\int_0^{1/2}\frac{1}{x^\alpha \log x}dx$$ exists finite. For me, this problem is very difficult. Any suggestions please?

0
Q: positive operator exercise

user119615I have a positive operator A on the Hilbert space $\mathcal{H}$. I must prove that $\|A\|=\sup_{x \ne 0}\frac{(Ax,x)}{(x,x)}$. I am only able to get one inequality: Assume x is nonzero: $\frac{(Ax,x)}{(x,x)}\le \|Ax\|\|x\|/\|x\|^2\le \|A\|$. So $\|A\|\ge\sup_{x \ne 0}\frac{(Ax,x)}{(x,x)}$. An...

Short title. Title contains exercise. positive operator exercise
0
Q: Rotation matrix check

display_errorLet matrix $A=\frac{1}{\sqrt{2}} \begin{bmatrix} 1 & -1 \\ 1 & 1 \\ \end{bmatrix} $. Check if $A$ is a rotation matrix in $\mathbb{R^2}$ by angle $\theta=\frac{\pi}{4}$. Entries of a matrix $A$ in trigonometric form are $A= \begin{bmatrix} \cos(-...

Short title. Tagged proof-verification. Rotation matrix check
 
11:59 AM
0
Q: Infinite telescoping series

EliminationConsider the following series: $$\sum_{n=1}^\infty \frac{\cos(nx(n-1) - \cos(nx+(n+1)}{2(n+x^2)}$$ Now, for every $n\ > 1$, we have cancellations as the series is telescoping. Having said that, would it be right to claim that the series equlas: $$\frac{1}{2x^2} + \frac{1}{2(1+x^2)}$$

0
Q: Finding the function of the power series $\sum\limits _{n=1}^{\infty}\frac{x^{2n+1}}{n}$

Nescio Find the sum function of the following power series $\sum\limits _{n=1}^{\infty}\frac{x^{2n+1}}{n}$ I actually felt like I had the right idea on this one, my solution is: Moving one $x$ out of the sum and then differentiating we get $$\sum\limits _{n=1}^{\infty}\frac{x^{2n+1}}{n}=x\sum\lim...

 
12:20 PM
0
Q: Is this series convergent? (which could not be solved by comparison theorems)

xlddLet $$c_n=\sum_{k=1}^{2n-1}\frac{1}{k^a}\left(\frac{1}{(2n-k)^a}-\frac{1}{(2n+1-k)^a}\right)$$ Can we show that $\sum_{n=1}^\infty c_n$ converge?

0
Q: Integral of dependent uniform RVs

wjk2a1Help please, If I have a: $X\sim U[0,a]$ s.t. $a>0$ $Y|X=x\sim U[0,x]$ then: $$f_Y (y)= \int_{-\infty}^{\infty} f_{X,Y}(x,y)dx = \int_{0}^{a} f_{Y|X}(y)\cdot f_X(x)dx=\int_{0}^{a} \frac{1}{ax} dx = \frac{1}{a}(ln(a)-ln(0))$$ How do I get past this? Thanks for any help!

Question contains please. Integral of dependent uniform RVs
0
Q: Is exponential of GUE random matrix Haar random?

Daniel RanardConsider the matrix exponential map $H \mapsto e^{i H t}$ acting on the Gaussian unitary ensemble (GUE) of Hermitian matrices. I would expect that for large $t$, the resulting measure on the unitary group approaches Haar measure -- is that right? Is there a simple heuristic argument showing tha...

Consider adding a tag for a broader subject area to which the question belongs. Some of these tags might fit. (from a bot)Normal Human 25 secs ago
0
Q: textbook on calculus of variation which focuses on the following topics

TaniI need a textbook (or set of online lecture notes) on calculus of variation which focuses on the following topics "Variation of a functional, Euler-Lagrange equation, Necessary and sufficient conditions for extrema , Variational methods for boundary value problems in ordinary and partial diff...

Tag (reference-request) should not be the only tag a question has. Please add a tag for a subject area to which the question belongs. (autocomment)Normal Human 21 secs ago
0
Q: A question on numerical range

H.SLet $A = \left( {\begin{array}{*{20}{c}} 0 & 0 & 0 & 1 \\ 0 & 0 & 1 & 0 \\ 0 & 0 & 0 & 0 \\ 1 & 0 & 0 & 0 \\ \end{array}} \right)$. What is numerical range of $A$?

Words such as question do not add information to titles. Please edit the title so that it better describes the specifics of your question. Do not hesitate to make it longer or include a formula if needed. More tips here. (from a bot)Normal Human 30 secs ago
0
Q: A positive real valued symetric matrix. Then A has only positive (not negative) Eigenvalues. determine if this is true for the following Matrices

CyrilA=| 3 sqrt(2) sqrt(6)| |sqrt(2) 1 sqrt(3)| | sqrt(6) sqrt(3) 4 | Hi im suppose to find Eigenvalues and eigen vectors for this Problem but when i find the det(A-xI) i get a complicated polynomial with imaginary numbers as roots. and then when i try to solve for the e...

Welcome to Math.SE, Cyril. This site uses MathJax formatting of formulas. More tips here. (from a bot)Normal Human 31 secs ago
0
Q: A question on concrete category

user45765This following is excerpted from Category Theory by S. Awodey. "Theorem 1.6. Every category C with a set of arrows is isomorphic to one in which the objects are sets and the arrows are functions." Remark 1.7. This shows us what is wrong with the naive notion of a “concrete” category of sets and ...

Words such as question are uninformative in titles. Please edit the title so that it better describes the specifics of your question. Do not hesitate to make it longer or include a formula if needed. More tips here. (autocomment)Normal Human 21 secs ago
 
12:59 PM
0
Q: Probable mistake at automated audit question

sadaf2605I had this at my first posts review queue as a automated audit test, I voted it "no action needed" and got blocked from reviewing for 1 month but I do not understand why this post should be voted otherwise. Here is the link of the question: http://stackoverflow.com/review/first-posts/10451996

 
0
Q: IMO 2014 problem 1

Nima BavariLet $a_0 < a_1 < a_2 < \cdots$ be an infinite sequence of positive integers. Prove that there exists a unique integer $n \geqslant 1$ such that $$ a_n < \frac {a_0 + a_1 + \cdots + a_n} {n} \leqslant a_{n + 1}. \qquad \qquad \qquad \qquad (1) $$

Short title. Title contains problem. IMO 2014 problem 1
0
Q: Exercise 6.79 from Rotman advance modern algebra

erfanIf $G$ is a nonzero abelian group show that $$Hom_{\mathbb Z}(G,\frac{\mathbb Q}{\mathbb Z}) \neq 0$$

Title contains exercise. Short question. Exercise 6.79 from Rotman advance modern algebra
0
Q: What is the relation between $R^{-1}$ and the set of units of $R$

user275240If $R$ is a ring. What is the relation between $R^{-1}$ and the set of units of $R$? Are they the same?

0
Q: Difficulty in solving calculus of variations problem.

zafranI am solving a problem on calculus of variation in which $F(x,y,y')$ is given as $F(x,y,y')=e^yy'^2$ After solving Euler equation I got this $2y'' +2y'-y'^2=0$. I don't know how to proceed further. Please guide me. Thanks in advance.

Title contains problem. Question contains please. Difficulty in solving calculus of variations problem.
0
Q: calculus - derivatives by definition of limits - help with problem

Itamar Silversteinquestion in calculus: f(x) = [x], show the derivative at point x=2 (by definition of limits) Thanx (:

Title contains help, problem. Short question. calculus - derivatives by definition of limits - help with problem
0
Q: Winding Number of a path

EyalAll definitions of a Winding Number I came across (or all equivalents), were only for a closed path. Could a Winding Number be defined for a path which is not closed? Will this definition be of any use?

 
 
1 hour later…
2:30 PM
0
Q: Find the maximum of $x$ satisfies the cubic equation

BlessLet $a,b,c,d$ be integers with $1\leq a,b,c,d\leq10$, and $x$ be real such that $ax^3-bx^2-cx-d=0$. How to find the maximum of $x$ ? Thanks in advance.

 
2:48 PM
0
Q: Limit of a multivariate function

Nimantha PriyamalI want to prove the following theorem. Theorem: If lim(x,y)→(a,b) f(x,y)=L and limx→a f(x,y)and limy→b f(x,y)exist then limx→a limy→b f(x,y)= limy→blimx→a f(x,y) =L. This is how I tried this. Assuming lim(x,y)→(a,b) f(x,y)=L and limx→a f(x,y)and limy→b f(x,y)exist, ∀ϵ>0,∃δ1>0;0<[(x-a)^2+(y...

 
-8
Q: Could you guys add a feature that when an answer is accepted, a tweet be published on behalf of the person answering the question?

The Sharp NinjaIt would be really nice if we could link out account to twitter and when an answer is accepted, the linked account would send a tweet like "My answer (short url) on #stackoverflowanswers was accepted by [user posting question]."

-2
Q: Tag synonym edit request: decryption is not encryption

ch3rub7I'm very new to SO community, and recently after answering this question about decryption, I wanted to edit the question by adding tag encryption and as while doing this to my surprise I found decryption and encryption as synonyms... here Well, I thought they were antonyms?!? Am I wrong? I read ...

 
This site uses MathJax formatting of formulas. More tips here. (autocomment)Normal Human 21 secs ago
0
Q: On the usage of results whose proof we did not read or did not understand

Ante PaladinThis question of mine is, if not completely then at least partially, motivated by the proposed proof of the $abc$ conjecture which was given by Mochizuki. Now suppose that I want to use consequences of the $abc$ conjecture in order to prove some other results and that I also did not read the pro...

Tag (soft-question) should not be the only tag a question has. Please add a tag for a subject area to which the question belongs. (from a bot)Normal Human 21 secs ago
0
Q: Notation in quantum groups.

LJRThe quantum group $U_q(sl_3)$ is generated by $E_1, E_2, F_1, F_2, K_1, K_2, K_1^{-1}, K_2^{-1}$ subject to some relations. I read some papers and there is a notation $K_{\lambda}$, where $\lambda$ is a weight. I think that we have $\lambda = a \alpha_1 + b \alpha_2$ for some numbers $a,b$. For e...

0
Q: Confusion about statistics

RileyI'm reading a paper on Truncated distributions, where some asymptotic tests are considered to determine if a data sample is truncated or not. I'm confused about the following line: An asymptotic test based on $L(\hat{\alpha})$ (this is a known statistic) rejects $H_0$ on asymptotic level $...

Short title. Title contains confus. Confusion about statistics
0
Q: Measurability of a function related to Skorohod space

DanielsenLet $(E,r)$ be a metric space, $D_E[0,\infty)$ be the Skorohod space on $[0,\infty)$ takes value in $E$. Consider the function $$(D_E[0,\infty)\times[0,\infty),\mathcal B(D_E[0,\infty)\times \mathcal B([0,\infty))\to (E,\mathcal B(E))$$ $$(x,t)\mapsto x_t$$ I am wondering whether this map is meas...

0
Q: basic question in inequalation

Silas2033$$ \frac{1}{x} <E $$ E always positive. how can I get to expression like this: $ x > something $ in 1 stage? if it's not possible in 1 stage (although this is what my teacher did) another answers is also good :)

Words such as question are uninformative in titles. Please edit the title so that it better describes the specifics of your question. Do not hesitate to make it longer or include a formula if needed. More tips here. (autocomment)Normal Human 21 secs ago
0
Q: is part inside loop at a vertex considered as region in graph theory? Edit

Sai CharanIs part inside loop at a vertex considered as region in graph theory? Edit

0
Q: Proving convergence of $\sum_{n=1}^\infty n^{ln(x)} $

MykyboI want to prove that $\sum_{n=1}^\infty n^{ln(x)} $ is convergent for x in $(0, \frac {1}{e})$ interval and divergent for $ x \geq \frac {1}{e} $. I am lost on how to prove it. Could someone please show me or give me a hint? I assume I would need to use one of the criterias for determining conv...

0
Q: Is $cos(\frac{pi}{x}$ uniformly continuous on (0,1)?

MateuszI have to use the definition of uniform continuity to disprove that $cos(\frac{pi}{x})$ is uniformly continuous, but I don't know how to do that.

0
Q: equal column sums of a matrix

user3701257Consider the non-negative matrix $B$ with dominant eigenvalue $\mu$. Let $s\ge\mu$. Let $R$ be the resolvent of $B$, i.e. $R=(sI - B)^{-1}$. What conditions must $s$ and $B$ satisfy such that all the column sums of $R$ are equal?

0
Q: How do I simplify this equation

Jesse the Wind WandererI'm trying to find a formula that will allow me to calculate the sum total of a progression (not sure if that's the word) in a spreadsheet. 1 + 0.79 + 0.79*0.79 + 0.79*0.79*0.79 + ... I can simplify the parts between the plus signs and graph them with 0.79^x But how do I calculate the sum of t...

Welcome to Math.SE, Jesse the Wind Wanderer. This site uses MathJax formatting of formulas. More tips here. (from a bot)Normal Human 21 secs ago
 
3:16 PM
0
Q: How can I make a .bat file run automatically when an alert box pops up?

Matthew WaiSometimes POP Peeper suddenly does not work on my computer. Whenever it happens, an alert box will pop up, and the box's PID in Task Manager is 244. The solution is to restart POP Peeper. I know how to use a .bat file to restart a program, but don't know how to make the file run automatically whe...

 
0
Q: $p$-Splittable Integers

Batominovski Let $p$ be a positive integer. For each nonnegative integer $k$, write $[k]$ for the set $\{0,1,2,\ldots,k\}$. Also, we define $[-1]:=\emptyset$. We say that an integer $k\geq -1$ is $p$-splittable if there is a partition of $[k]$ into $p$ subsets $A_1$, $A_2$, $\ldots$, $A_p$ such that $\s...

0
Q: Krylov Matrix Tridiagonal Decomposition

AccioHogwartsI am reading through "Matrix Computations" by Gene H. Golub and Charles F. Van Loan and have come across a proof on the properties of Tridiagonal Decomposition that seems to gloss over parts I do not understand. The Theorem states that: If $Q^TAQ=T$ is the tridiagonal decomposition of symmetr...

Tagged proof-explanation. Krylov Matrix Tridiagonal Decomposition
 
0
Q: Is it wrong to answer a question that others are discussing in the comments?

zagadka314I am referring to this question: basic question in inequation I started to answer the question, but as I wrote my answer, someone was commenting with the OP and I changed my answer as new information came to light (instead of two cases, only one, so just deleted one case). It is obviously a sim...

 
3:34 PM
0
Q: IMO 2007 problem 1

Nima BavariReal numbers $a_1, a_2, \cdots, a_n$ are given. For each $i$ ($1 \leqslant i \leqslant n$) define $$ d_i = \max \{a_j : 1 \leqslant j \leqslant i \} - \min \{a_j : i \leqslant j \leqslant n \} $$ and let $$ d = \max \{d_i : 1 \leqslant i \leqslant n\}. $$ (a) Prove that, for any real numbers $x_...

Tag (contest-math) should not be the only tag a question has. Please add a tag for a subject area to which the question belongs. (autocomment)Normal Human 21 secs ago
0
Q: what is the sum of the multiplication table?

s.a.a1 2 3 4 ...10..n n 2n 3n ...100..n^2 Blockquote what is the sum of this table? i think that it maybe solvable with series. i'll be grateful if anyone could help me.

0
Q: Finding extrema with lagrange moltipliers

darril12I'm trying to find the extrema of f(x,y)= cos(x²-y²) constrained to x²+y²=1. Using Lagrange Multipliers I get this far: -x(sin(2x²-1)=λx -y(sin(-2y²+1)=λy But I don't know how to proceed after this. Could anyone explain to me what I have to do next please?

Welcome to Math.SE, s.a.a. Tag (contest-math) should not be the only tag a question has. Please add a tag for a subject area to which the question belongs. (from a bot)Normal Human 21 secs ago
0
Q: Something fishy in the Zeta function

MasanoriRecently I came across the Riemmann representation of the Zeta function as follows: Zeta(s) = (2^s)*(pi^s-1)*sin(pi*s/2)*Gamma(1-s)*Zeta(1-s) Now, I went ahead to calculate the term Gamma(1-s) for s=2, knowing beforehand the values of the otehr terms, namely, Zeta(2) = pi^2/6 (Bassel problem)...

Welcome to Math.SE, Masanori. This site uses MathJax formatting of formulas. More tips here. (autocomment)Normal Human 21 secs ago
 
-1
Q: More -rep when downvoting answers where you answered?

bytecode77I have a suggestion Often, people down vote answers on questions where they themselves answered, just so their own answer is further on top. This has nothing to do with the answer being bad, so it's considered strategically down voting. Sometimes they remove their down votes as long as they can...

 
0
Q: Solving a system of ODE (3 equations) using runge-kutta method order 4

ChrisKx' = λ - ρx - βxz; y' = βxz - δy; z' = py - cz; x0=43100; y0 = 0, z0 = 0.0033, λ = 388, ρ = 0.009 δ = 0.18, p = 50000, c = 23, β=3.61e-8 my code is as follows: f = @(t,x) [388-0.009*x(1)-0.0000000361*x(1)*x(3); 0.0000000361*x(1)*x(3) - 0.18*x(2); 50000*x(3) - 23*x(3)]; tspan = 0:7:84; [t,xa] = o...

Welcome to Math.SE, ChrisK. This site uses MathJax formatting of formulas. More tips here. (from a bot)Normal Human 21 secs ago
0
Q: Find out the number of solutions to the equation- x1+x2+x3+x4+x5+x6=25x1+x2+x3+x4+x5+x6=25 under certain constraints

Brandon TurpyHow many solutions are there to the equation x1 + x2 + x3 + x4 + x5 + x6 = 25 where each xi is a non-negative integer and 3 ≤ x1 ≤ 10

This site uses MathJax formatting of formulas. More tips here. (autocomment)Normal Human 21 secs ago
0
Q: Big O Notation Clarification

markthethomasWorking through a textbook on algorithms (CLRS intro to algorithms) and just wanted to see if someone could help me understand one of the exercises at the end of a chapter. Problem: Is n^(2+1) = O(n^2)? The book uses = loosely to convey the "set of" notation, so it's more like "is n^(2+1) con...

This site uses MathJax formatting of formulas. More tips here. (autocomment)Normal Human 21 secs ago
0
Q: gradient of the gradient !!

hichamgaussfor $u$ scalaire function defined on $\mathbb{R}^n$ we have this equality : $\nabla(|\nabla u|^{p-2})=(p-2)|u|^{p-4}|\nabla u|^{p-4}\nabla u\nabla\nabla u$ my question is : what doese it mean $\nabla\nabla u$ because i know only the signification of the gradient of a function , but here will be ...

0
Q: Proving convergence of $\sum_{n=1}^\infty cos(n^2\pi)\cdot(\sqrt{n+11}-\sqrt{n+2}) $

MykyboI want to prove that $\sum_{n=1}^\infty cos(n^2\pi)\cdot(\sqrt{n+11}-\sqrt{n+2}) $ is convergent (or divergent). I am lost on how to prove it. Could someone please show me or give me a hint? I assume I would need to use one of the criterias for determining convergence? I tried to multiplicate ...

0
Q: Prey predator excercice

dudundeeConsider the 2-dimensional system of non-linear ODEs, semplified instance of a predator-prey population model $\dot x=\alpha x (1-x)-xy$ $\dot y = y(x-y)-\beta y$ with $\alpha = 1 ,\beta = 1/2$ Why does $x(0) > 0$ and $y(0) > 0$ imply that $x(t) > 0$ and $y(t) > 0$ for all times (populations...

0
Q: let p be a prime number. sort groups of order $p^2$

gpgpgpI have this question i need to solve - let p be a prime number. sort groups of order $p^2$ Any help or hint that will help me begin solving this question will help.

 
4:00 PM
-1
Q: Add comment not work

Lonly.wolfWhen i use opera mini 4.5 on mobile view not full site and then click "add comment" to responding question reply, it look like no thing happen at all, the small reply box didn'nt show up

 
0
Q: Having difficulties understanding this stretched string greens function problem.

ChickenblAshtI'll start by stating the problem text: "A string is streched along the x-axis from $x=0$ to $x= \pi/2$ and is made to vibrate by a force proportional to - $f(x)sin( \omega t)$. The amplitude, $y(x)$ , of small vibrations is then given as the solution to: $$ y'' + y = f(x) $$ Where $y(0) = y(\pi...

Title contains problem. Tagged pde, differential-equations. Having difficulties understanding this stretched string greens function problem.
0
Q: Integration Identity problem

RSparkesBy making the substitution $x=\pi -t$ show that; $$\int_{0}^{\pi} xf(\sin x)dx = \frac{1}{2}\pi \int_{0}^{\pi} f(\sin x)dx$$ where $f(\sin x) $ is a given function of $\sin x$. I get $dx = -dt$ and that $\sin (\pi -t) = \sin t$ which gives me; $$\int_{0}^{\pi} xf(\sin x)dx =\int_{0}^{\pi} tf(\s...

Short title. Title contains problem. Integration Identity problem
 
0
Q: All Questions of all favourite tags

HarisI have been answering question that has the c tag from a long time. Recently I started learning scheme and thought to go through some incoming questions which has the scheme tag. But then I realized I have to keep 2 tabs open if I want to see the questions of both the tags. Isn't there a way to ...

 
0
Q: How does this question apply to mine?

Jean-ClaudeA little earlier I posted a question Isomorphism between ring of polynomial functions on unithyperbola and Laurent polynomials which was marked as duplicate by somebody who didn't want to help me further. He said that my question is a duplicate of Localization in a ring However I don't see wh...

Words such as question do not add information to titles. Please edit the title so that it better describes the specifics of your question. Do not hesitate to make it longer or include a formula if needed. More tips here. (autocomment)Normal Human 21 secs ago
0
Q: $\mathbb{C} = \mathbb{R}[t]/(t^2 + 1)\mathbb{R}[t]$

flipbackAt a lecture about factor rings the lecturer gave us the following example: $$\mathbb{C} = \mathbb{R}[t]/(t^2 + 1)\mathbb{R}[t]$$ He said that it was quite obvious and skipped the explanation. But it is not obvious for me at all and I have some questions: How could $(t^2 + 1)\mathbb{R}[t]$ ...

A title should not be all-MathJax; having some plain text helps with search and navigation. (from a bot)Normal Human 21 secs ago
0
Q: Partial derivatives chain rule help understanding

courageuxIn this example here: I am having trouble understanding the steps in applying the product rule, like what is set as u, du, v, du? Please help clarify.

0
Q: How to prove algebraic multiplicity is less than or equal to geometric multiplicity

ChammySo the algebraic multiplicity is largest power k such that (x-lambda)^(k) which is a root of the characteristic polynomial. Geometric multiplicity is the dimension of an eigenspace for each lambda. Eigenspace= Ker(T-lambda.I) I set a basis for this eigenspace to be {v_1,...,v_m) I extended the...

Words such as help are uninformative in titles. Please edit the title so that it better describes the specifics of your question. Do not hesitate to make it longer or include a formula if needed. More tips here. (autocomment)Normal Human 24 secs ago
This site uses MathJax formatting of formulas. More tips here. (autocomment)Normal Human 21 secs ago
 
4:18 PM
0
Q: Synonymize [tomee] and [apache-tomee]

Didier LI was just reviewing suggested edits and I saw several edits that consisted in changing tags from apache-tomee to tomee because the former has the following excerpt: DO NOT USE THIS TAG! Please use [tomee] instead. apache-tomee has almost 200 questions. tomee has almost 300 questions. Th...

0
Q: Soliciting for upvotes?

GertIn the comments to this question I advised that giving a full answer to a HW&E question is not what we should do. The question which contains nothing of conceptual value, is clearly a HW question and isn't well formatted, appears eligible for 'close' votes (but I haven't voted to close yet). Af...

 
0
Q: order relations & doubts

user284901It is given a relation R={(1,1); (2,2); (3,3); (4,4); (5,5); (3,5); (4,3); (4,2); (4,5); (2,5)} for a set A={1, 2, 3, 4, 5}. I found this relation is reflexive, anti-symetric and transitive, i.e an order relation. If order relations exists, there must be unique "factor" that is valuable for all...

0
Q: Homeomorphic ( Toplogy)

Ahmad Talafha would please someone help me out with that. I print Screen for everything I have done so far. The assumption is missing .

Short title. Short question. Question contains please. Homeomorphic ( Toplogy)
0
Q: IMO 2007 problem 5

Nima BavariLet $a$ and $b$ be positive integers. Show that if $4ab - 1$ divides $(4a^2 - 1)^2$, then $a = b$.

Short title. Title contains problem. Short question. IMO 2007 problem 5
0
Q: Asymptote to sin x/x?

Master ShurikenI have seen elsewhere that: $y=\sin x/ x$ has a horizontal asymptote of $x=0$, as it approaches that line as x tends to +/- infinity. Now, why does it not have an asymptote at $y=0$ or $x=1$, as the curve tends towards but never touches these lines? (Which satisfies the definition given by wol...

0
Q: Using rsolve in Maple

JamgreenI have tried using rsolve in Maple to obtain a recursion formula from an ordinary differential equation with summations. I get Is there some reason for Maple not calculating the sums? It seems rsolve is not used at all; however, it seems to work if I don't use sums in $x(t)$ and $u(t)$. I a...

Short title. Question contains please. Tagged maple. Using rsolve in Maple
0
Q: How to write a grammar that accepts the language formed by strings $e^{3n} f g{^2n} h$?

RedBird$e^{3n}\: f \: g^{2n}\: h$ n ∈ ℕ∪{0} For Example: fh eeefggh eeeeeefggggh I am stuck at the $e^{3n}$ and $g^{2n}$ part. This is what I have so far: S→EfGh E→eE∣ϵ G→gG∣ϵ Please correct me if I am wrong. Thanks

0
Q: How many cuts are needed to divide a circle into $N$ pieces?

user4201961How many cuts are needed to divide a circle into $N$ pieces (not necessarily equal pieces)? Is there any formula for this?

 
4:35 PM
-1
Q: Why has my question [Polygonal Mersenne numbers] been put on hold

redelectronsWhy has my question Polygonal Mersenne numbers been put on hold. I understand that at first it was unclear what my question was, but I think that now it is (clear). Could someone please put it of hold, or if it still needs improving, tell me if I should specify the question even more (not sure ho...

 
0
Q: augmented matrix for a linear system

coOoraBika Hawi**Let enter image description here be the augmented matrix for a linear system. For what values of a and b does the system have: a) A unique solution; b) Infinitely many solutions; c) No solutions.

 
0
Q: I improved an old, downvoted, already answered question. How can I attract potential upvoters?

Coert GrobbelaarIn my earlier days of Stack Overflow I asked this question. My goal was to learn more about SQL injection, and the reasoning behind the established SQL best practices. Due to a lack of question writing experience on my part, the question did not reflect my goal clearly, and people thought I was ...

 
0
Q: Product topology with upper and lower limits

bvt5050Consider X = ℝ with the lower limit topology, and Y = ℝ with the upper limit topology. The set [0,1) × [0,1) is an open set in the product topology on X × Y.

0
Q: Simple question on the closure of a differential operator

user3482534Consider $A:\mathcal{D}(A)\subset L^{2}[0,1]\to L^{2}[0,1]$ given by $$A:=-\frac{d^{2}}{dx^{2}},\qquad\mathcal{D}(A):=C^{2}_{0}(0,1)$$ Now, I assume that the closure of $A$ is its extension defined on $C^{2}_{0}[0,1]$. Then am I right in thinking that this is this densely defined if $\overline{C...

Words such as question are uninformative in titles. Please edit the title so that it better describes the specifics of your question. Do not hesitate to make it longer or include a formula if needed. More tips here. (from a bot)Normal Human 21 secs ago
 
4:54 PM
0
Q: Have ability to message edit proposers when reviewing their edits

JenBRelated to Message writer of well meaning but unnaceptable edits Instead of putting a public comment on the question, it would be great to be able to privately message a <2000 rep user who has proposed edits with a message like 'Thank you for improving this post, but please make sure you do all t...

 
0
Q: Difference between R^{4} and $^{1,3}

failexamWhat's the difference between $R^{4}$ and $R^{1,3}$? I know that the first one has metric Kronecker delta $\delta_{ij}$. Does the second one have Minkowski metric $g_{\mu \nu}$?

0
Q: order relations & doubts

user284901It is given a relation R={(1,1); (2,2); (3,3); (4,4); (5,5); (3,5); (4,3); (4,2); (4,5); (2,5)} for a set A={1,2,3,4,5}. I found this relation is reflexive, anti-symetric and transitive, i.e. an order realation. If order relation exists, there must be unique factor that is valuable for all orde...

0
Q: logics and proofs

srishti77714this question in given in discreet mathematics by kenneth rosen. Each inhabitant of a remote village always tells the truth or always lies. A villager will only give a "yes " or a "No" response to a question a tourist asks. Suppose you are a tourist visiting this area and come to a fork in the r...

Short title. logics and proofs
0
Q: Normed Quotient Space

shoe burtIf $X$ is a normed vector space and $M$ is a proper closed subspace, I want to show that for any $\epsilon>0$ there exists an $x\in X$ such that $\|x\|=1$ and $\|x+M\|\geq 1-\epsilon$. Is there anything wrong with arguing as follows: Suppose by contradiction there exists some $\epsilon >0$ such ...

0
Q: Neumann problem with inhomogenous Laplace equation

rootI have the following problem, and I don't know what I'm missing: Let $\Omega = D(0,1)\setminus\{0\}$. I want to find all $f \in C^2(\Omega)$, such that $$\Delta_{\Bbb R^2} f = \frac{\partial^2 f}{\partial x^2} + \frac{\partial^2 f}{\partial y^2} = r \cos\varphi = g(r,\varphi) \qquad (x,y...

0
Q: Extended Euclidean Algorithim Question

Aceboy1993I get the Extended Euclidean Algorithm but I don't understand what this question is asking me: In this question we consider arithmetic modulo 60 ("clock arithmetic" on the clock with numbers {0,1,2,3,...,58,59}). Which of the following statements apply? a. 50 X 50 = 40 modulo 60 b. 17 + 50 =...

Words such as question do not add information to titles. Please edit the title so that it better describes the specifics of your question. Do not hesitate to make it longer or include a formula if needed. Questions tend to get more attention when they have a tag for a broad area of mathematics relevant to the question. Some of these tags might fit. More tips here. (from a bot)Normal Human 21 secs ago
0
Q: Questions on Interpretation and Semantic

Krishna KalyanHomework Question We have been given the homework question that is in the picture. I am not sure if we understand correctly what is asked of us. Would be grateful for any help.

0
Q: Pointwise convergence

darrenIs this series convergentor divergent. I first thought it was convergent due to pointwise limits existing at every value of x but now im not sure wether or ot I am right. My series is fn(x)=(x^n)/(1+x^n) with x ∈ [0,1]. How do I decide wether this series is convergent or divergent.

Consider replacing (analysis) with a more specific tag for the relevant branch of analysis. (autocomment)Normal Human 20 secs ago
1
Q: Explaining a proof of Euler's theorem

WhykaCan someone please explain the questionmarked extrapolation in the following image?

Questions tend to get more attention when they have a tag for a broad area of mathematics relevant to the question. Some of these tags might fit. (autocomment)Normal Human 21 secs ago
 
5:16 PM
0
Q: Mirror site with a funny name?

user96343I came across this site http://math.dingding.in/ while I was looking up a user online. I am just curious how the above link leads to the same site(I mean the web address seems a bit unconventional.) Just curious.

 
0
Q: Mapping preserves implications

user285146Let $S$ and $T$ be sets, and let ${\phi}:S\, {\rightarrow}\,T $ be a mapping. Let $*$ be an operation on $S$. Let $x$ and $y$ be any elements of $S$ and let $a$ be any of the set's cancellable elements (under $*$). Then, from the above, we have that ${\forall}x,y,a\,{\in}\,S:x\,*\,a=y\,*\,a{\im...

0
Q: Hints on exersice in functional analysis

User2313I solved a) and b). But I cant seem to get a grip of what characterizes the functionals for which we want continuity in the general case. Hints please!

Short question. Question contains please. Hints on exersice in functional analysis
Words such as interesting are uninformative in titles. Please edit the title so that it better describes the specifics of your question. Do not hesitate to make it longer or include a formula if needed. More tips here. (from a bot)Normal Human 20 secs ago
0
Q: Motivation for dual bases

Kevin ShengI am encountering dual bases for the first time in the context of algebraic number theory, mainly in proofs regarding the existence of an integral basis for $\mathcal{O}_K$ and its ideals. I am wondering about motivation for defining a dual basis and intuitively what it is. So far I have been wor...

0
Q: b is the inverse of a (mod 11)

Jose LLet a and b be numbers in the set S = {0, 1, 2, 3, 4, 5, 6, 7, 8 , 9, 10} such that b is the inverse of a (mod 11) and a and b are not equal. How many such subsets {a, b} of S are there?

0
Q: An interesting diophantine equation

Shalabh SinghLet there be a natural number $a,$ that can be expressed as $a = x^p - y^p$ where $x, y$ and $p$ are natural numbers, each two of them being pairwise co-prime and $p$ is an odd prime. Then can $x$ and $y$ be found out uniquely in terms of $a$ and $p$? If yes, how?

Words such as interesting do not add information to titles. Please edit the title so that it better describes the specifics of your question. Do not hesitate to make it longer or include a formula if needed. More tips here. (from a bot)Normal Human 20 secs ago
 
5:50 PM
0
Q: Trouble finding a sequence formula

That1guyThe sequence goes: 1,$\frac{2}{3}$,$\frac{7}{9}$,$\frac{20}{27}$,$\frac{61}{81}$ I tried using the common difference method of analysis and found the second row follows the rule: multiply by $\frac{-1}{3}$, Third row onwards seems to follow the rule: multiply by $\frac{1}{3}$. I can't figure ou...

Title contains troubl. Trouble finding a sequence formula
0
Q: Bivariate normal distribution proving and finding parameters

GanjiraI have $X$ and $Y$ which are independent random variables following the normal distribution. How should I prove that a random variable ($Y-2X$, $X+3Y$) has bivariate normal distribution? And how should I find the values of parameters for it? Thanks in advance!

Questions tend to get more attention when they have a tag for a broad area of mathematics relevant to the question. Some of these tags might fit. (from a bot)Normal Human 21 secs ago
 
00:00 - 18:0018:00 - 00:00

« first day (33 days earlier)      last day (541 days later) »